PT48.S1.Q20 - Researcher: We have found that some cases

Small_victoriesSmall_victories Free Trial Member
edited March 2019 in Logical Reasoning 104 karma

Hello!
This NA question is causing my blood pressure to rise.
I have trouble accepting E).
Negating E) doesn't destroy the argument. It's okay if medicine DOES reduce stress, as long as it isn't ONLY reducing stress. Maybe it reduces stress and inhibits hormone production.

I bring this up because in the second premise, it says that "any illness caused by stress--> treatable ONLY by the reduction of stress".
To bridge the gap between this premise and the conclusion, we need a NA that states "Medicine does not treat high blood pressure ONLY by reduction of stress", NOT "Medicine used to treat HBP does not reduce stress", which is what E) is saying.

I feel like if the premise had said, "any illness caused by stress--> treatable by the reduction of stress", AC E) would be absolutely the necessary assumption of the argument.

Please help. I'm so stuck :'( :'(
Thank you!!

Admin note: added link
https://7sage.com/lsat_explanations/lsat-48-section-1-question-20/

Comments

  • Logic GainzLogic Gainz Alum Member
    edited November 2018 700 karma

    Hi @Small_victories ! Apologies on the length of my explanation in advance!

    At first I agreed with your analysis and was going to trump this up to holding this test to too high a standard, but now I see where your reading went wrong!

    I think you're mischaracterizing the conditional statement beginning with, "since it is generally accepted." In other words, you're using, "is", to be the necessary condition indicator when you should be using, "only", as the necessary condition indicator.

    You might be reading the statement as below:

    [Illness caused by stress] => [treatable only by the reduction of stress]

    In the above statement, you're using "is" to be the necessary condition indicator. Now let's read it again, but this time we'll make, "only", be the necessary condition indicator which is how I think the argument intends it to be:

    [Illness caused by stress is treatable] => [by the reduction of stress]

    The conclusion negates the sufficient condition above and we, the test taker, have to get there by negating the necessary condition. In doing that we need to say the medication doesn't reduce stress at all and that's what E gets us.

    It seems like it's too strong, but that's how this stimulus is set up in my opinion.

  • rationalbeingrationalbeing Free Trial Member
    edited November 2018 19 karma

    We need to connect the first sentence to the last part of the argument. The clear gap is that the conclusion doesn't mention medicine, but it alludes to the first sentence that talks about some cases of high blood pressure that can be treated effectively with medicine.

    So medicine must treat high blood pressure without reducing stress. Otherwise, you cannot conclude that some cases must not be caused by stress.

  • Small_victoriesSmall_victories Free Trial Member
    edited November 2018 104 karma

    Thank you Logic Gainz for taking your time to understand my confused state of mind! I was worried no one would comprehend my confusion.

    I have a question about the conditional statement representation:

    [Illness caused by stress is treatable] => [by the reduction of stress]

    If you take the contrapositive of this statement, you get:

    [NOT by reduction of stress]--> [Illness caused by stress is NOT treatable]

    I am slightly uncertain about this conditional representation because the the conclusion states that "Some cases of HBP must not be caused by stress". It doesn't conclude that some cases of HBP are not treatable. Since the conclusion doesnt match the contrapositive 100%, I went with [Illness caused by stress] => [treatable only by the reduction of stress].

    Im not sure though!

    Your comment really helped me realize that I actually misunderstood the meaning of "ONLY".

    The necessary assumption is actually that "medicine's treatment of HBP does not necessarily reduce stress".

    I originally wrote that negating E doesnt wreck the argument because medicine may reduce stress and inhibit hormone production. My logic was that negating E allows for the possibility that medicine could reduce stress AND inhibit hormones.

    This is mistaken. I confused what's meant by "requiring stress reduction" with "stress reduction being the ONLY stand-alone method" of treating stress-caused illnesses. I thought that "treatable ONLY by reduction of stress" meant no combination of stress reduction and other method was allowed.

    E makes much more sense when I understand it to mean that "Medicine does not necessarily reduce stress while treating HBP". Do I make sense at all?

  • Logic GainzLogic Gainz Alum Member
    700 karma

    Yeah this is tough... the good news is that I think I follow your reasoning.

    Given your original interpretation, we could have medicine that does, in fact, reduce stress, but it also treats the stress-caused illness via another way like hormone inhibition. Therefore, the real necessary assumption would say, "...the medication for high blood pressure works by more ways than just stress reduction." We don't need the medication to not reduce stress at all. We just can't have stress reduction as its only means to combating high blood pressure.

    Hmm... I think the contrapositive/conclusion mismatch can be rectified if we do something I hate to admit to doing because it sounds uber friggin technical, but we have to push illness caused by stress into the enigmatic "domain".

    I'm sure you've seen some of JY's videos in which he'll use a pseudo-venn diagrammatic approach when solving a question where he'll draw a circle and label the circle something which indicates a set of something. I wish I was there with you to draw this out, but imagine I draw one of those circles and it stands as the set of something and I label that circle, "Stress causing Illnesses". Now if we're in the domain of, "stress-causing illnesses", or practically speaking, in that circle, we know that we have a conditional statement: "Treatable => reduction of stress". Every time we violate that conditional statement, we know we aren't in the domain of "stress-causing illnesses" any longer, because in that world/domain, that conditional statement must hold.

    So, because HBP can be treated with medication, and we have to include our "necessary assumption" of medicine not reducing stress, we are then violating that conditional statement by having - abstractly speaking, A => B, but yet we have A (HBP treatable) while having ~B (no stress reduction.

    This blatant violation of this conditional statement indicates that we are no longer in the domain of stress-causing illnesses, and therefore we have our match. Jeeeeeeesus I hate this explanation. I don't even know if it makes sense to me, but that's what I have at the moment. Again, sorry for the length.

  • Small_victoriesSmall_victories Free Trial Member
    104 karma

    Thank you @"Logic Gainz" ! I totally get what you mean by the domain. Thank you for explaining it so thoroughly. I think I understand the question and the AC's now!!

Sign In or Register to comment.